Difference between revisions of "2018 AMC 10A Problems/Problem 1"

(Step by step solution)
m
Line 14: Line 14:
 
<cmath> =\frac{11}{7} </cmath>
 
<cmath> =\frac{11}{7} </cmath>
 
Therefore, the answer is <math>\boxed{\textbf{(B) } \frac{11}{7} }</math>.
 
Therefore, the answer is <math>\boxed{\textbf{(B) } \frac{11}{7} }</math>.
 +
 +
==Video Solution==
 +
https://youtu.be/vO-ELYmgRI8
  
 
== See Also ==
 
== See Also ==

Revision as of 06:13, 21 January 2020

Problem

What is the value of \[\left(\left((2+1)^{-1}+1\right)^{-1}+1\right)^{-1}+1?\] $\textbf{(A) } \frac58 \qquad \textbf{(B) }\frac{11}7 \qquad \textbf{(C) } \frac85 \qquad \textbf{(D) } \frac{18}{11} \qquad \textbf{(E) } \frac{15}8$

Solution

\[\left(\left((2+1)^{-1}+1\right)^{-1}+1\right)^{-1}+1\] \[=\left(\left(3)^{-1}+1\right)^{-1}+1\right)^{-1}+1\] \[=\left(\left(\frac{1}{3}+1\right)^{-1}+1\right)^{-1}+1\] \[=\left(\left(\frac{4}{3}\right)^{-1}+1\right)^{-1}+1\] \[=\left(\frac{3}{4}+1\right)^{-1}+1\] \[=\left(\frac{7}{4}\right)^{-1}+1\] \[=\frac{4}{7}+1\] \[=\frac{11}{7}\] Therefore, the answer is $\boxed{\textbf{(B) } \frac{11}{7} }$.

Video Solution

https://youtu.be/vO-ELYmgRI8

See Also

2018 AMC 10A (ProblemsAnswer KeyResources)
Preceded by
First Problem
Followed by
Problem 2
1 2 3 4 5 6 7 8 9 10 11 12 13 14 15 16 17 18 19 20 21 22 23 24 25
All AMC 10 Problems and Solutions

The problems on this page are copyrighted by the Mathematical Association of America's American Mathematics Competitions. AMC logo.png